LSAT and Law School Admissions Forum

Get expert LSAT preparation and law school admissions advice from PowerScore Test Preparation.

 Administrator
PowerScore Staff
  • PowerScore Staff
  • Posts: 8916
  • Joined: Feb 02, 2011
|
#23164
Complete Question Explanation

Flaw in the Reasoning. The correct answer choice is (E)

In this stimulus the conclusion of the argument in the last line states that the critics were right in their evaluation. Looking back into the stimulus, we find that the critics condemned the sale of the art because losing "first-rate pieces" violates the museum's duty as a trustee of art. In order for the conclusion to follow, then, the pieces of art must be shown to be first rate. We know that the director considered these works inferior, but the argument uses the evidence of cost to prove their value. There is basic causal reasoning present here: the cause of the high price is the quality of the works. However, the stimulus also states that the paintings were sold in "an otherwise stagnant art market," which could mean that the market conditions dictated the high price of the art and not the fact that they were "first rate." This is a classic causal Flaw question.

Answer choice (A) This answer is trying to tempt you because of the differences in opinion between the director and the critics. However, it concludes that the art is first rate not because of the number of critics but because of the high price.

Answer choice (B) We do not know who is a more qualified expert here, the director or the art critics, so this answer choice does not follow. Furthermore, similar to answer choice (A), the argument is not based on judgment or opinion. It is based on the price of the art.

Answer choice (C) The stimulus is not rejecting any "proven means" that the museum may be employing. The conclusion is about the correctness of the critics' evaluation; it is not a rejection of the "means" of the director.

Answer choice (D) The "firm conclusion" is based on the present price of the art, not on any future price or "state."

Answer choice (E) This is the correct answer choice, because the argument presupposes that the "first-rate" quality of the art caused the high prices when it could have been the "stagnant" art market.
 Sdaoud17
  • Posts: 85
  • Joined: Apr 13, 2013
|
#9412
can you explain more how the stimulus has a causal flaw. When I approach I thought its a future /past price flaw because Its says that after months from selling the Paining from the museum. It was sold 3 times more. :-? :-?

Thank you
 Ron Gore
PowerScore Staff
  • PowerScore Staff
  • Posts: 220
  • Joined: May 15, 2013
|
#9413
Thank you for your question, Sdaoud17!

You were definitely on the right track in your approach to this question, focusing on the time shift element of the stimulus. Carrying that idea a bit further in your analysis would have brought you to the causal flaw described in Answer (E). So, let's look at how to make that next step.

Two events occurred in the stimulus: 1) the Museum Director sold the paintings for one amount; and 2) the same paintings were resold for two to three times that amount. You correctly point out that there was a time shift of a few months between the first and the second set of sales.

A common reasoning flaw on the LSAT is the time-shift error, in which it is assumed by the author that conditions are materially the same at the time of the second event as they were at the time of the first event, or vice versa. We often see this time-shift error occur in the context of scientific theories or inferences from archeological finds, cases in which the intervening period is significantly longer. We also see time-shift errors occur when the author attempts to predict the future based on past or even current circumstances.

Here, despite the relatively small shift of only a few months, the stimulus expressly tells us a detail about the market at the time of the second set of sales that we didn't have for the first set, i.e., the art market was otherwise stagnant.

Now, the LSAT cannot assume we have an economics background, and answering this question correctly cannot depend on your knowing precisely what impact a stagnant art market would have on the second set of sales. However, the LSAT does expect us to recognize that the circumstances have changed, and that this change may have some effect on the art sale. Notice that we are talking here about causality.

Other than mentioning in passing the stagnant art market, the passage does not expressly deal with its existence or its effects when siding with the Critics over the Museum Director. Without justification, the stimulus author has selected from a pool of causal options only one potential cause, the undervaluation of the art by the Museum Director, as the cause for the differing sales prices. Failing to deal with the stagnant art market at the time of the second sale as a potential alternate cause leading to the effect of a change in the sales price of the art is a logical flaw.

Answer (E) describes this flaw, and is correct for that reason.

Also, understand that there are many other potential causes for the difference in the sales price of the art at issue. It is always a risky business on the LSAT to observe some state of affairs and then to infer what caused it. There often are several competing potential causes that may have worked independently, or perhaps even in combination, to bring about the observed effect.

Hope that helps!

Ron
 est15
  • Posts: 94
  • Joined: Aug 28, 2013
|
#15505
For this question, I thought the flaw in the argument was that the critics used a future event (paintings resold at high prices) to justify their present claim (paintings are first-rate). So I thought the answer was D. Can you explain what's wrong with D and why E is correct? Is it in fact valid reasoning to use something in the future to justify something in the present?

Thanks.
 Steve Stein
PowerScore Staff
  • PowerScore Staff
  • Posts: 1153
  • Joined: Apr 11, 2011
|
#15517
Hi,

That's a good question. In that one, a museum director is criticized for selling paintings that are, some critics claim, first rate--although the museum director considers the works inferior.

Based on the fact that the paintings were soon sold for two to three times the price paid in the original sale, the author concludes that the critics were right when they claimed that the paintings were indeed first rate.

In review, the museum director felt the paintings were inferior, but the critics felt they were first-rate.

But does the increase in price really prove that the paintings are first-rate? The price increase could have been caused by something else, such as a surge in the art market; this is basically what correct answer choice (E) provides: The author bases a conclusion on the price increases, even though those increases could have been caused by something else.

I hope that's helpful! Please let me know whether this is clear--thanks!

Steve
 est15
  • Posts: 94
  • Joined: Aug 28, 2013
|
#15533
Hi Steve,

Thanks for your reply. I now understand why E is correct but for why D is wrong, is that because generally and not just for this problem it's valid to use future evidence to justify something in the present?
 Steve Stein
PowerScore Staff
  • PowerScore Staff
  • Posts: 1153
  • Joined: Apr 11, 2011
|
#15536
Hi,

Thanks for your response. Answer choice (D) would be an issue, but in this case, the critics don't base a firm conclusion about the present on speculative claim about the future. Instead, they claim that their first assertion (that the works were first-rate) were later proven true by subsequent sale prices. That questionability of that proof is the issue in this case.

I hope that's helpful! Please let me know whether this is clear--thanks!

Steve
User avatar
 annabelle.swift
  • Posts: 54
  • Joined: Sep 01, 2021
|
#92249
I was between A and E on this question.

I thought A was correct because the author concludes that a certain opinion (that the paintings are first-rate) is correct on the grounds that it is held by more people (the critics and the buyers of the paintings who paid 2-3x the price paid to the museum) than hold the opposing view (the museum director). Can you explain why this thinking is incorrect?
 Adam Tyson
PowerScore Staff
  • PowerScore Staff
  • Posts: 5153
  • Joined: Apr 14, 2011
|
#92267
The problem with answer A, annabelle.swift, is that the author is not resting their argument on the number of people who hold an opinion. It's not "more people think this than that, so this is true" (a flawed appeal to popular opinion/numbers). No, their evidence is that "these prices settle the issue." The argument rests on the idea that the high selling prices prove that the critics were right that the pieces were first-rate that the museum should not have let go. The problem is that the prices might not prove anything about the quality of the art! Maybe they sold for high prices simply because the market was stagnant, and buyers were desperate to get any new art, even art that is of lower quality. Or maybe they sold for high prices because the buyers were fooled into thinking they were better than they are. Or maybe there was some other cause.

It wouldn't matter to the author which side had more supporters; as far as they are concerned, the selling prices are their proof regardless of how many people said one thing or the other. Thus, A is describing the wrong grounds for reaching the conclusion.
User avatar
 sjlsat
  • Posts: 8
  • Joined: Jan 24, 2023
|
#100053
Hello,

I picked answer choice D for this one after I was stuck between D and E. I now know why E is correct, but I want to make sure I really understand why D is wrong and what would have made D right so I can break down these answers choices better.

If the conclusion said something like “Clearly, these prices show that the museum has lost first rate pieces and therefore negatively impacted art for future generations,” would D be right? If not, what is an example of a conclusion that would have made D right? Thank you!

Get the most out of your LSAT Prep Plus subscription.

Analyze and track your performance with our Testing and Analytics Package.